0 Daumen
3,1k Aufrufe

Aufgabe:

Z.z

(1-1/n^2)^n=1

Hinweis: Bernolische Ungleichung


Problem/Ansatz:

Meine Idee ist: (1-1/n^2)^n=1 als ((1-1/n)*(1+1/n))^n =1 zu schreiben und da ich weiß, dass (1+1/n)=e ist (wir haben letzte Woche diese bewiesen) kann ich (1-1/n)= -e sagen?  und e*(-e)=1?

Avatar von

Hast du die 4) mittlerweile verstanden?

nee... ich habe noch nicht angefangen. Bin heute sehr faul...

die ist wirklich selten einfach, man muss bei vielen gar nicht nachdenken.

okai.. sieht aber nicht so aus. bist du schon damit fertig?

Jo, habs gemacht.

Brauche deine Hilfe :)ich

Klick auf mein Profil, dort findest du meine E-Mail-Adresse. Andernfalls stelle deine Fragen bitte ein. Hier in der Kommentarsektion macht das wenig Sinn.

Vom Duplikat:

Titel: Grenzwert beweisen mit der Bernoullischen Ungleichung

Stichworte: analysis,grenzwert,ungleichungen

Aufgabe:

Zeigen Sie:

\( \lim\limits_{n\to\infty} \) (1-\( \frac{1}{n^2} \))n  =1

Problem/Ansatz:

Als Hinweis hatten wir die Bernoullische Ungleichung bekommen, ich habe mich daran versucht aber bin mir sehr unsicher ob ich das richtig gemacht habe.

Erstmal habe ich folgendes gemacht:

1-\( \frac{1}{n^2} \) = \( \frac{n^2}{n^2} \) - \( \frac{1}{n^2} \) = \( \frac{n^2-1}{n^2} \)

\( \frac{n^2-1}{n^2} \) ≤ 1 für n gegen ∞

Also ist auch (1-\( \frac{1}{n^2} \))n ≤ 1n (Hier habe ich einfach beide Seiten der Ungleichung hoch n genommen)

Bernoullische Ungleichung:

x∈(-1,∞)\{0} und n∈ℕ, n>1 gilt:

(1+x)n ≥ 1+nx

Für x habe ich -\( \frac{1}{n^2} \) gewählt, für p=1+x und x=p-1

pn ≥ 1+n(-\( \frac{1}{n^2} \))

pn ≥ 1-\( \frac{1}{n} \)

==> 1-\( \frac{1}{n} \) ≤(1-\( \frac{1}{n^2} \))n ≤ 1n


Ist meine Herangehensweise richtig oder habe ich einen Fehler gemacht?


Grüße und danke für die Aufmerksamkeit

2 Antworten

+2 Daumen
 
Beste Antwort

(1 - 1/n^2)^n = 1


(1 + x)^n ≥ 1 + n·x mit x = -1/n^2

(1 - 1/n^2)^n ≥ 1 + n·(-1/n^2)

(1 - 1/n^2)^n ≥ 1 - 1/n

für n --> ∞ ergibt sich daher

(1 - 1/n^2)^n ≥ 1 - 0

Eine Zahl etwas kleiner als 1 kann hoch unendlich aber nie größer als 1 werden oder?

Also gilt auch

(1 - 1/n^2)^n ≤ 1

Und damit letztendlich auch

(1 - 1/n^2)^n = 1

wohlgemerkt für n → unendlich das ich nicht immer dazuschreibe.

Avatar von 477 k 🚀

Super!!! Eigentlich ganz simpel.

schön! vielen Dank

+1 Daumen

den Tipp mit der Bernoullischen Ungleichungen verstehe ich nicht. Allerdings kannst du das Einschließungskriterium verwenden:$$e^{\frac{1}{n}}\geq \left(1-\frac{1}{n^2}\right)^n\geq 1-\frac{1}{n}$$

Die Grenzwerte der äußeren Grenzen sollte klar sein?!

Avatar von 28 k

ja,  den Tipp verstehe ich auch nicht. Das stand beim Aufgabenblatt so.


danke für deine Antwort.

Ich weiß, ich muss das auch bearbeiten ;-). Wir hatten mal gezeigt, dass für alle k in den ganzen Zahlen lim ^n√n^k=1 für n gegen unendlich. Also ist auch im ^n√e^1=1 für n gegen unendlich.

1+1/n ist klar. 1/n ist eine Nullfolge und dann bleibt halt noch die Konstante Zahl 1.

Also haben beide den Grenzwert eins, nach dem Enschließungskriterium muss dann auch der Ausdruck in der Mitte gegen 1 laufen.... OK?

:)

hast du die 4 schon?

Die Folge ist falsch.

Nein, bei der juckt es mich nicht so sehr in den Fingern - aber die ist doch über die Definition heute leicht machbar ....

zebra :-)

Die Folge ist falsch.

Du schon wieder :( - bitte direkt mit Einwand.

Ein Vorzeichen ist falsch.

bei der Aufgabenstellung?

Nein, in meiner Antwort. Habe den Tippfehler nun ersetzt.

Dann stimmt die Ungleichung nicht mehr.

Jetzt gibt es glaube ich keine Einwände mehr...

Ein anderes Problem?

Stell deine Frage

Willkommen bei der Mathelounge! Stell deine Frage einfach und kostenlos

x
Made by a lovely community